Question

Calculate the torque (magnitude and direction) about point O due to the force F⃗  in each of the cases sketched in the figure (Figure 1). In each case, the force F⃗  and the rod both lie in the plane of the page, the rod has length 4.00 m, and the force has magnitude 16.0 N .

Constants Part C Calculate the torque (magnitude and direction) about point O due to the force F in each of the cases sketchePart E Constants Calculate the magnitude of the torque in case (c) Calculate the torque (magnitude and direction) about pointout of the page to the right upward Constants Calculate the torque (magnitude and direction) about point O due to the force F

0 0
Add a comment Improve this question Transcribed image text
Answer #1

Torque direction of torque is given by right hand rule Case a) part (A) 0 Part B direction out of the page Case b)Part c 0 20 F- 16 N angle between vector I and F #2 180-120 60. t IFsin 60 t-4 x16x sin 60 -32 x V3N m 32 x 3 55.426N m Part0 3o τ = 1 Fsin 30 τ=4×16×sin 30 = 32 τ=32N·m Part F direction out of the page Case d) Part G) 60 0τ--Fsin 60 τ 2x16xsin 60 16 x V3 Part H direction into the page Case e) Part-l 60° 0 τ=0 Part Jline of action of force passes through the point oso the torque is zero Case f) Part K) Part L line of action of force pass

Add a comment
Know the answer?
Add Answer to:
Calculate the torque (magnitude and direction) about point O due to the force F⃗  in each of...
Your Answer:

Post as a guest

Your Name:

What's your source?

Earn Coins

Coins can be redeemed for fabulous gifts.

Not the answer you're looking for? Ask your own homework help question. Our experts will answer your question WITHIN MINUTES for Free.
Similar Homework Help Questions
  • Constants Part A Calculate the torque magnitude and direction) about point O due to the force...

    Constants Part A Calculate the torque magnitude and direction) about point O due to the force F in each of the cases sketched in the figure (Figure 1). In each case, the force F and the rod both lie in the plane of the page, the rod has length 4.00 m, and the force has magnitude 15.0 N You may want to review (Pages 303 - 306) For related problem-solving tips and strategies, you may want to view a Video...

  • Exercise 10.1Calculate the torque (magnitude and direction) about point O due to the force F in...

    Exercise 10.1Calculate the torque (magnitude and direction) about point O due to the force F in each of the cases sketched in the figure (Figure 1) . In each case, the force F and the rod both lie in the plane of the page, the rod has length 4.00 m, and the force has magnitude 17.0 N. Part E Calculate the magnitude of the torque in case (c). |tau|= N.m Part F Find the direction of the torque in case...

  • Calculate the torque (magnitude and direction) about point O due to the force F?

    Calculate the torque (magnitude and direction) about point O due to the force F? in each of the situations sketched in the figure below. In each case, the force F? and the rod both lie in the plane of the page, the rod has length 4.00 m , and the force has magnitude 12.0 N . Let counterclockwise torques be positive.Calculate for cases a-f:

  • Calculate the torque (magnitude and direction) about point OOO due to the force F in each...

    Calculate the torque (magnitude and direction) about point OOO due to the force F in each of the situations sketched in the figure below (Figure 1). In each case, the force Fand the rod both lie in the plane of the page, the rod has length 6.00 m, and the force has magnitude 5.00 N. Let counterclockwise torques be positive. Figure < 1 of 1 > 0 900 120.0 (b) 60.00 30.0 2.00 m 60.00 0 o (1) Part A...

  • Constants Part A A square metal plate 0.180 m on each side is pivoted about an...

    Constants Part A A square metal plate 0.180 m on each side is pivoted about an axis through point o at its center and perpendicular to the plate (Figure 1). You may want to review (Pages 303 - 306) For related problemsolving tips and strategies, you may want to view a Video Tutor Solution of Applying a torque. Calculate the net torque about this axis due to the three forces shown in the figure if the magnitudes of the forces...

  • Part A A square metal plate 0.180 m on each side is pivoted about an axis...

    Part A A square metal plate 0.180 m on each side is pivoted about an axis through point O at its center and perpendicular to the plate (Figure 1) Calculate the net torque about this axis due to the three forces shown in the figure if the magnitudes of the forces are F 27.0N, F2 -16.4 N, and F3 15.2 N. The plate and all forces are in the plane of the page. Take positive torques to be counterclockwise You...

  • Calculate the net torque (magnitude and direction) on the beam in the figure below about the...

    Calculate the net torque (magnitude and direction) on the beam in the figure below about the following axes. Assume a = 46°,8 = 31°, Y = 23, 4.2 m) (a) an axis through O perpendicular to the page magnitude Nm direction Select (b) an axis through C perpendicular to the page magnitude D N .m direction Select

  • Calculate the net torque (magnitude and direction) on the beam about the following axes. 6. (-/1...

    Calculate the net torque (magnitude and direction) on the beam about the following axes. 6. (-/1 Points) DETAILS SERCP7 8.P.003. Calculate the net torque (magnitude and direction) on the beam in Figure P8.3 about the following axes. 25 N 809 200 45° 2.0 m 10 N 4.0 m 30 N Figure P8.3 (a) an axis through o, perpendicular to the page Nom - Direction (b) an axis through C, perpendicular to the page Nom Direction...

  • Calculate the net torque (magnitude and direction) on the beam in the figure below about the...

    Calculate the net torque (magnitude and direction) on the beam in the figure below about the following axes. 25 N 30° 200 45°〉 | 2.0 m 10 N 30 N (a) an axis through O perpendicular to the page magnitude direction N·m counterclockwise (b) an axis through C perpendicular to the page magnitude N·m counterclockwise

  • Calculate the net torque (magnitude and direction) on the beam in the figure below about the...

    Calculate the net torque (magnitude and direction) on the beam in the figure below about the following axes. (Assume α = 43°, β = 28°, γ = 16°, ℓ = 3.6 m) (a) an axis through O perpendicular to the page (b) an axis through C perpendicular to the page

ADVERTISEMENT
Free Homework Help App
Download From Google Play
Scan Your Homework
to Get Instant Free Answers
Need Online Homework Help?
Ask a Question
Get Answers For Free
Most questions answered within 3 hours.
ADVERTISEMENT
ADVERTISEMENT
ADVERTISEMENT